Newspaper subscriber: Arnot's editorial argues that by making certain fundamental changes in government we would virt...

kbernard on July 2, 2020

Answer choice B?

Why not B?

Reply
Create a free account to read and take part in forum discussions.

Already have an account? log in

shunhe on July 2, 2020

Hi @kbernard,

Thanks for the question! So let’s recap the stimulus real quick first. A newspaper subscriber is telling us that Arnot’s editorial argues that changing government would eliminate social ills. But, says the newspaper subscriber, Arnot’s wrong. And the newspaper subscriber believes this because Arnot’s argument depends on a questionable assumption.

So now we’re asked to figure out what most accurately expresses a flaw in the argument’s reasoning. And clearly, what’s wrong here is the fact that the author goes from “you haven’t proven your argument” to “your conclusion is false.” There’s a gap between those two things, and that’s the heart of the matter that (A) gets at. (B) just doesn’t do it. It tells us that the argument treats a change that’s required for eliminating social ills as a change that will guarantee the virtual elimination; in other words, it confuses a necessary condition and a sufficient condition. But that’s not what happens here since there’s not even really conditional logic at play. Besides, the argument never says that the change is required for eliminating social ills. So (B) can’t be the right answer here.

Hope this helps! Feel free to ask any other questions that you might have.